SchemesTap - January 2023 - Quiz-Complete-Test Lyst9135

You might also like

Download as pdf or txt
Download as pdf or txt
You are on page 1of 71

3/29/23, 10:06 AM SchemesTap - January 2023 - Quiz-complete-test

SchemesTap - January 2023 - Quiz


Government Schemes
Q.1) Which of the following statements is/are incorrect with respect to the 'Agriculture Infrastructure
Fund' scheme?
A. It is Centrally Sponsored Scheme.

B. It was launched in the year 2021.

C. The scheme has been continued up to 2029-30.

[A] A and B

[B] A and C

[C] B and C
[D] A, B and C

Q.2) Which of the following statements is/are correct with respect to the 'Agriculture Infrastructure
Fund'?

A. Rs. 2 Lakh Crore will be provided by banks and financial institutions as loans to eligible beneficiaries.

B. Interest subvention of 4% per annum up to a limit of loans of Rs. 2 Crore.


C. Moratorium for repayment may vary subject to minimum 6 months and maximum of 18 months.

[A] A and B

[B] A and C

[C] B and C

[D] None of the above

Q.3) Which among the following is not one of the beneficiaries under the 'Agriculture Infrastructure
Fund'?

[A] Farmers, Farmer Producer Organizations, Primary Agricultural Credit Society

[B] Marketing/Multipurpose Co-operative Societies, SHGs, Joint Liability Groups

[C] Agri-entrepreneurs, Start-ups

[D] All of the above are beneficiaries

Q.4) What is the tenure of the 'Atal Mission for Rejuvenation and Urban Transformation 2.0'?

https://edutap.learnyst.com/admin/products/all-questions/385022 1/71
3/29/23, 10:06 AM SchemesTap - January 2023 - Quiz-complete-test

[A] 2020-21 to 2023-24

[B] 2021-22 to 2024-25

[C] 2020-21 to 2027-28


[D] 2021-22 to 2025-26

Q.5) What is the total financial outlay under the 'Atal Mission for Rejuvenation and Urban
Transformation 2.0' scheme?

[A] Rs. 2,99,000 crore

[B] Rs. 2,88,000 crore

[C] Rs. 2,77,000 crore


[D] Rs. 2,66,000 crore

Q.6) Which of the following statement(s) is/are correct with respect to 'Atal Mission for Rejuvenation
and Urban Transformation 2.0' scheme?

A. The total amount of Central funding is Rs. 76,760 crore.

B. For North-Eastern and Himalayan States, 80% funding is provided by central government.
C. Cities having million plus population are mandated to take up PPP projects worth minimum of 25%
of their total project fund allocation.

[A] A only

[B] B only

[C] C only
[D] A and C

Q.7) With respect to the Promotion of  Farmer Producer Organizations (FPOs)  scheme, identify the
incorrect statement:
A. It is a centre sponsored scheme.

B.   It has been set up with Rs.16,000 Crores under AtmaNirbhar Bharat Abhiyaan Stimulus Package.

C.      The Central Government is providing credit guarantee of 25% of total borrowing for MSME projects.

[A] Only A

[B] Only B

[C] Only C

[D] Both A and B

https://edutap.learnyst.com/admin/products/all-questions/385022 2/71
3/29/23, 10:06 AM SchemesTap - January 2023 - Quiz-complete-test

Q.8) The central government will provide ___ % interest subvention under the  Promotion of  Farmer
Producer Organizations (FPOs) scheme.

[A] 1%
[B] 2%

[C] 3%

[D] 4%

Q.9) Which of the following statements is correct with respect to the Promotion of Farmer Producer
Organizations (FPOs) scheme?

[A] 3 years moratorium period for principal amount & 6 years repayment period will be provided.
[B] Credit Guarantee Fund of Rs. 750 Crores is set up by government and managed by NABARD.

[C] The Nodal Ministry is Department of Animal Husbandry and Dairying

[D] Individual entrepreneur are not the beneficiaries under the scheme.

Q.10) Which of the following statements is/are correct with respect to 'Atal Bhujal Yojana'?

A. It is being implemented by Ministry of Rural Development.


B. It was launched in the year 2018.

C. The scheme will be implemented for 5 years from 2019-20 to 2023-24.

[A] A and B

[B] A and C

[C] B and C
[D] None of the above

Q.11) What is the total outlay of 'Atal Bhujal Yojana' scheme?

[A] Rs. 2000 Crore

[B] Rs. 6000 Crore

[C] Rs. 4000 Crore

[D] Rs. 5000 Crore

Q.12) Which of the following statements is/are correct with respect to 'Atal Bhujal Yojana'?

A. It is Centrally Sponsored Scheme.

https://edutap.learnyst.com/admin/products/all-questions/385022 3/71
3/29/23, 10:06 AM SchemesTap - January 2023 - Quiz-complete-test

B. The scheme aims at bringing about behavioural change at the community level through awareness
programs and capacity building for fostering sustainable ground water management.

C. The scheme has 2 components, incentive component and institutional strengthening and capacity
building component. 
[A] A and B

[B] A and C

[C] B and C

[D] A, B and C

Q.13) 'Atal Bhujal Yojana' is being supported by the _________ on a ________ basis funding along with
Government of India?

[A] IMF, 60:40

[B] WTO, 40:60

[C] ADB, 50:50

[D] World Bank, 50:50

Q.14) Which of the following statements is/are correct with respect to the objectives of the
Compensatory Afforestation Fund Management and Planning Authority (CAMPA)?

A. To promote afforestation and development activities.

B. To recommend measures based on strategic planning to the authorities of the State CAMPA.

C. To resolve issues that arise between inter-state or Centre-State.


[A] A and B

[B] A and C

[C] B and C

[D] A, B and C

Q.15) Who is the Chairman of the Compensatory Afforestation Fund Management and Planning
Authority (CAMPA) at the National level?
[A] President of India

[B] Prime Minister

[C] Union Minister of Environment & Forests

[D] State Minister for Environment and Forests

https://edutap.learnyst.com/admin/products/all-questions/385022 4/71
3/29/23, 10:06 AM SchemesTap - January 2023 - Quiz-complete-test

Q.16) What is the percentage of funds collected under the above mentioned scheme to be transferred
to the State CAMPAS?

[A] 90%
[B] 30%

[C] 50%

[D] 70%

Q.17) Which of the following statements is/are correct with respect to the Credit Guarantee Fund Trust
for Micro and Small Enterprises (CGTMSE) scheme?

A. Educational / Training Institutions, SHG and agriculture are eligible for coverage under the scheme.
B. The corpus of CGTMSE is being contributed by the GoI and SIDBI in the ratio of 1:1 respectively.

C. Fund and non-fund based (Letters of Credit, Bank Guarantee etc.) credit facilities up to Rs 500 lakh
per eligible borrower are covered under the guarantee scheme.

[A] A and B

[B] A and C
[C] B and C

[D] None of the above

Q.18) In which year was the  Credit Guarantee Fund Trust for Micro and Small Enterprises (CGTMSE)
launched?

[A] 2015
[B] 2000

[C] 2019

[D] 2020

Q.19) Which of the following statements is/are correct with respect to the guarantee coverage under
the Credit Guarantee Fund Trust for Micro and Small Enterprises (CGTMSE)?

A. The extent of guarantee cover is 75% for micro enterprises for credit up to Rs 5 lakh.
B. The extent of guarantee cover is 50% of the sanctioned amount of the credit facility for credit from
Rs 10 lakh to Rs 100 lakh per MSE borrower.

C. The extent of guarantee cover is 80% for Micro and Small Enterprises operated and/or owned by
women.

[A] A and B

https://edutap.learnyst.com/admin/products/all-questions/385022 5/71
3/29/23, 10:06 AM SchemesTap - January 2023 - Quiz-complete-test

[B] A and C

[C] B and C

[D] A, B and C

Q.20) Which of the following statements is/are correct with respect to the Digital Health Incentive
Scheme (DHIS)?

A. Scheme has been launched by National Health Authority (NHA).

B. It was launched in the year 2022.

C. It aims to give a further boost to digital health transactions in the country under the Ayushman
Bharat Digital Mission (ABDM).
[A] A and B

[B] A and C

[C] B and C

[D] A, B and C

Q.21) The Digital Health Incentive Scheme (DHIS)  offers incentives of up to _____ to hospitals,
diagnostic labs and digital health solution providers such as Hospital/ Health Management Information
System (HMIS) and Laboratory Management Information System (LMIS).

[A] Rs. 1 crore

[B] Rs. 2 crores

[C] Rs. 3 crores


[D] Rs. 4 crores

Q.22) As per Digital Health Incentive Scheme (DHIS), which one of the following entity is not eligible to
receive incentives?

[A] Health Facilities having 10 or more beds

[B] Laboratory/radiology diagnostics centres

[C] Digital Solution Companies (entities providing ABDM enabled digital solutions)
[D] All of the above are eligible

Q.23) Which of the following are the programmes under the Environment Education, Awareness and
Training (EEAT) scheme?

A. National Green Corps “Ecoclub” programme.


https://edutap.learnyst.com/admin/products/all-questions/385022 6/71
3/29/23, 10:06 AM SchemesTap - January 2023 - Quiz-complete-test

B. Capacity Building Activities

C. National Nature Camping programme.


[A] A and B
[B] A and C

[C] B and C

[D] A, B and C

Q.24) In which year was the Environment Education, Awareness and Training (EEAT) scheme launched?

[A] 1973-74

[B] 1983-84
[C] 1993-94

[D] 2003-04

Q.25) Which of the following statements is/are correct with respect to the Fast Track Special Court
Scheme?

A. It is being implemented by Department of Justice.


B. The financing of the Scheme will be on the pattern of Centrally Sponsored Scheme.

C. Setting up Fast Track Courts (FTCs) and its functioning comes within the domain of the State
Governments who set up such courts in consultation with respective High Courts.

[A] A, B and C

[B] A and C
[C] B and C

[D] A, and B

Q.26) In which year was the Fast Track Special Court Scheme launched?

[A] 2015

[B] 2017

[C] 2018
[D] 2019

Q.27) What is the target number of Fast Track Special Courts (FTSCs) that will be set up by the
government?

https://edutap.learnyst.com/admin/products/all-questions/385022 7/71
3/29/23, 10:06 AM SchemesTap - January 2023 - Quiz-complete-test

[A] 700

[B] 897

[C] 984
[D] 1023

Q.28) The estimated size of the 'Fisheries and Infrastructure Development Fund' is Rs ______ crore and
the Nodal Loaning Entities are expected to raise Rs _____ crore.

[A] 7522, 5266

[B] 9522, 7466

[C] 5266, 1316


[D] 9522, 1316

Q.29) Which of the following statements is/are correct with respect to the 'Integrated Management of
Public Distribution System (IMPDS)' scheme?

A. It involves use of advanced data analytics techniques to bring about continuous improvements in
PDS operations.
B. It is a Central Sector Scheme.

C. National level de-duplication of all ration cards/beneficiaries’ data will also be taken up under the
scheme.

[A] A and B

[B] A and C
[C] B and C

[D] A, B and C

Q.30) In which year was the 'Integrated Management of Public Distribution System (IMPDS) scheme?

[A] 2015

[B] 2016

[C] 2017
[D] 2018

Q.31) Which of the following statements is/are correct with respect to Atal Vayo Abhyuday Yojana
(AVYAY)?

A. Ministry of Social Justice & Empowerment is the nodal ministry for the scheme.
https://edutap.learnyst.com/admin/products/all-questions/385022 8/71
3/29/23, 10:06 AM SchemesTap - January 2023 - Quiz-complete-test

B. Department of Social Justice and Empowerment is the implementing authority of the scheme.

C. It is a Central Sector Scheme.


[A] A and B
[B] A and C

[C] B and C

[D] A, B and C

Q.32) Under the Atal Vayo Abhyuday Yojana (AVYAY), upto ____ of the cost of the project is provided by
Government of India, while remaining shall be borne by Organization/ Institution concerned.

[A] 60%
[B] 50%

[C] 75%

[D] 90%

Q.33) Which of the following statements is/are correct with respect to ‘Jal Shakti Abhiyan:Catch the
Rain’?
A. Theme - “Catch the rain, where it falls, when it falls”.

B. It covers only rural areas of all districts of the country.

C. Setting up of Jal Shakti Kendras and intensive afforestation are part of the campaign.

[A] A and B

[B] A and C
[C] B and C

[D] A, B and C

Q.34) In which year was the ‘Jal Shakti Abhiyan:Catch the Rain’ scheme launched?

[A] 2021

[B] 2017

[C] 2019
[D] 2020

Q.35) Which among the following is the nodal agency for the ‘Jal Shakti Abhiyan:Catch the Rain’?

[A] National Water Mission

https://edutap.learnyst.com/admin/products/all-questions/385022 9/71
3/29/23, 10:06 AM SchemesTap - January 2023 - Quiz-complete-test

[B] KVIC

[C] NABARD

[D] TRIFED

Q.36) Which of the following statements is/are correct with respect to the Jan Shikshan Sansthan
scheme?

A. Currently, it is being implemented by Ministry of Skill Development and Entrepreneurship.

B. It is funded through 100% annual grant from the Government of India.

C. This scheme is implemented through NGOs.

[A] A and B
[B] A and C

[C] B and C

[D] A, B and C

Q.37) Which among the following are part of the new guidelines issued in 2019 under the 'Jan Shikshan
Sansthan'?
A. Evidence-based assessment system

B. Centralization of powers for JSSs

C. Linking JSS to Public Finance Management system

[A] A and B

[B] A and C
[C] B and C

[D] A, B and C

Q.38) Which among the following is not one of the scope of work under the 'Jan Shikshan Sansthan'
scheme?

[A] Develop appropriate curriculum and training modules.

[B] Training equivalent to courses designed by the Directorate of Adult education, National Institute of
Open Schooling and Director General Employment & Training.

[C] Provide training to a pool of resource persons and master trainers for conducting training.

[D] All of the above

https://edutap.learnyst.com/admin/products/all-questions/385022 10/71
3/29/23, 10:06 AM SchemesTap - January 2023 - Quiz-complete-test

Q.39) Which of the following statements is/are correct with respect to the 'Make in India' scheme?

A. Department of Commerce coordinates action plans for service sectors.

B. It was launched in the year 2015.


C. Department for Promotion of Industry and Internal trade (DPIIT) coordinates action plans for
manufacturing sectors.

[A] A and B

[B] A and C

[C] B and C

[D] A, B and C

Q.40) Which among the following is not one of the pillars under the 'Make in India' scheme?

[A] New Processes

[B] New Technologies

[C] New Mindset

[D] New Sectors

Q.41) Which among the following are the targets under the 'Make in India' scheme?

[A] Target an increase in manufacturing sector growth to 12-14% per annum over the medium term.

[B] An increase in the share of manufacturing in the country’s Gross Domestic Product from 16% to
33% by 2022.

[C] To create 50 million additional jobs by 2022 in manufacturing sector


[D] 2 & 3 only

Q.42) Which of the following statements is/are correct with respect to the Mission for Integrated
Development of Horticulture (MIDH) scheme?

A. Under MIDH, Government of India (GOI) contributes 60%, of total outlay for developmental
programmes in all the states except states in North East and Himalayas.

B. It was launched in the year 2014.


C. In the case of North Eastern States and Himalayan States, GOI contributes 75%.

[A] A and B

[B] A and C

[C] B and C

https://edutap.learnyst.com/admin/products/all-questions/385022 11/71
3/29/23, 10:06 AM SchemesTap - January 2023 - Quiz-complete-test

[D] A, B and C

Q.43) Consider following statements regarding the Mission for Integrated Development of Horticulture
(MIDH) and choose the correct ones?
A. It is a Central Sector Scheme.

B. This scheme is a sub-scheme of Green Revolution - Krishonnati Yojana.

C. National Horticulture Mission is one of the sub-mission of the discussed scheme.

[A] A and B only

[B] B and C only

[C] A and C only


[D] A, B and C

Q.44) Which among the following is not one of the sub-schemes under the 'Mission for Integrated
Development of Horticulture (MIDH)'?

[A] Horticulture Mission for North East and Himalayan States (HMNEH)

[B] National Horticulture Board (NHB)


[C] Coconut Development Board (CDB)

[D] All of the above

Q.45) Which of the following statements is/are correct with respect to 'Mission Organic Value Chain
Development for North Eastern Region (MOVCDNER)'?

A. It aims to create at least one or two replicable end-to-end organic value chain models in every State.
B. It is a sub-mission under National Mission for Sustainable Agriculture (NMSA).

C. It is a Centrally Sponsored Scheme.

[A] A and B

[B] A and C

[C] B and C

[D] A, B and C

Q.46) In which year was the 'Mission Organic Value Chain Development for North Eastern Region
(MOVCDNER)' launched?

[A] 2006

https://edutap.learnyst.com/admin/products/all-questions/385022 12/71
3/29/23, 10:06 AM SchemesTap - January 2023 - Quiz-complete-test

[B] 2009

[C] 2013

[D] 2015

Q.47) Which among the following is not one of the states for which the 'Mission Organic Value Chain
Development for North Eastern Region (MOVCDNER)' is not applicable?

[A] Sikkim

[B] Assam

[C] Nagaland

[D] All of the above

Q.48) Which of the following statements is/are correct with respect to the 'Mission POSHAN 2.0'?

A. It is being implemented by Ministry of Women and Child Development.

B. It was launched in the year 2020.

C. Mission POSHAN 2.0 is an Integrated Nutrition Support Programme.

[A] A and B
[B] A and C

[C] B and C

[D] A, B and C

Q.49) Which of the following statements is/are correct with respect to the funding pattern between
centre and states in 'Mission POSHAN 2.0'?
A. 75:25 for States and UTs with legislature.

B. 90:10 for NER & Himalayan States and UT of J&K.

C. 100% for Union Territories without legislature.

[A] A and B

[B] A and C

[C] B and C
[D] A, B and C

Q.50) Which among the following is not part of Mission POSHAN 2.0?

[A] Anganwadi Services


https://edutap.learnyst.com/admin/products/all-questions/385022 13/71
3/29/23, 10:06 AM SchemesTap - January 2023 - Quiz-complete-test

[B] One Stop Centres

[C] Poshan Abhiyaan

[D] National Creche Scheme

Q.51) Which of the following statements is/are correct with respect to the 'Mission Shakti'?

A. It is being implemented by Ministry of Women and Child Development.

B. It consists of 2 sub schemes - Sambal and Samarthya.

C. Mission Shakti is approved for duration of 15th Finance Commission.

[A] A and B

[B] A and C
[C] B and C

[D] A, B and C

Q.52) Which among the following is not part of Sambal, a sub-scheme under 'Mission Shakti'?

[A] One Stop Centres

[B] Women Helplines


[C] Beti Bachao Beti Padhao

[D] Working Women Hostels

Q.53) Which among the following is not part of Samarthya, a sub-scheme under 'Mission Shakti'?

[A] Ujjwala

[B] Swadhar Greh


[C] Women Helplines

[D] Working Women Hostels

Q.54) Which of the following statements is/are correct with respect to the Modified Special Incentive
Package Scheme (M-SIPS) scheme?

A. It allows for Capital Subsidy of 25% for investments in Special Economic Zones (SEZs).

B. Incentives will be provided only for new units.


C. Incentives for a period of 3 years from the date of approval of application will be provided.

[A] A and B

https://edutap.learnyst.com/admin/products/all-questions/385022 14/71
3/29/23, 10:06 AM SchemesTap - January 2023 - Quiz-complete-test

[B] A and C

[C] B and C

[D] None of the above

Q.55) In which year was the Modified Special Incentive Package Scheme (M-SIPS) scheme launched?

[A] 1998

[B] 2002

[C] 2006

[D] 2012

Q.56) In which year was the Museum Grant Scheme launched?


[A] 2006

[B] 2009

[C] 2013

[D] 2015

Q.57) What is the maximum amount of financial assistance per project which may be given under the
Museum Grant Scheme for normal states?

[A] 50%

[B] 60%

[C] 70%

[D] 80%

Q.58) Which of the following are the components under the Museum Grant Scheme?

A. Development and Establishment of Museums at the regional, state and District level.

B. Digitization of Museum Collections.

C. Capacity building and training of Museum Professionals.

[A] A and B

[B] A and C
[C] B and C

[D] A, B and C

https://edutap.learnyst.com/admin/products/all-questions/385022 15/71
3/29/23, 10:06 AM SchemesTap - January 2023 - Quiz-complete-test

Q.59) What is the number of Urban Forests that have been targeted under the Nagar Van Yojana
(NVY)?

[A] 100
[B] 400

[C] 300

[D] 200

Q.60) In which year was the Nagar Van Yojana (NVY) launched?

[A] 2015

[B] 2017
[C] 2019

[D] 2020

Q.61) Warje Urban Forest will be considered as a role model for Nagar Van Scheme. This forest is
located in _________.

[A] Pune, Maharashtra


[B] Badlapur,Maharashtra

[C] Lonavala, Maharashtra

[D] Nagpur, Maharashtra

Q.62) With respect to the National Action for Mechanised Sanitation Ecosystem (NAMASTE) Scheme,
identify the incorrect statement.
[A] The scheme was launched in 2022

[B] The total budget allocated for the scheme is Rs. 360 crores

[C] The tenure of the scheme is from 2021-2026

[D] A NSKFDC is the implementing agency of the scheme

Q.63) Under the National Action for Mechanised Sanitation Ecosystem (NAMASTE)  Scheme, the
funding support in form of fixed rate of interest and subsidy for new ventures will be provided to the
beneficiaries. With regards to this information, identify the livelihood assistance that will be provided
to the beneficiaries under this scheme.

A. The rate of interest chargeable for the project upto 1 Lakh is 5% for women.

B. Upfront Capital subsidy for individuals from 5 Lakh to 15 Lakh is 50% of the project cost.
https://edutap.learnyst.com/admin/products/all-questions/385022 16/71
3/29/23, 10:06 AM SchemesTap - January 2023 - Quiz-complete-test

C. Repayment Period for the projects costing above 5 Lakh is 5 years with 6 months of moratorium
period included.
[A] A and C
[B] B and C

[C] A and B

[D] None of the above

Q.64) With reference to the National Apprenticeship Promotion Scheme, which of the following
statements is/are incorrect?

1. NAPS has replaced the erstwhile Atal Bihari Protsahan Yojana.


2. It covers all categories of apprentices including the Graduate and Technician apprentices.

3. Courses approved by State Government/Central Government shall be linked with apprenticeship


training.

[A] A only

[B] B only
[C] C only

[D] A and B only

Q.65) What is the maximum percentage of share of stipend to be provided by the employer under the
National Apprenticeship Promotion Scheme?

[A] 25
[B] 20

[C] 40

[D] 30

Q.66) Which of the following statements is/are correct with respect to the National AYUSH Mission
(NAM)?

A. It is being implemented by Ministry of Ayush.


B. It was launched in the year 2015.

C. It aims to build on India’s unmatched heritage represented by ancient systems of medicine.

[A] A and B

[B] A and C

[C] B and C
https://edutap.learnyst.com/admin/products/all-questions/385022 17/71
3/29/23, 10:06 AM SchemesTap - January 2023 - Quiz-complete-test

[D] A, B and C

Q.67) Which among the following is not one of the flexible components of the National AYUSH Mission
(NAM)?
[A] AYUSH Wellness Centres comprising Yoga and Naturopathy

[B] IEC (Information, Education and Communication) activities

[C] Voluntary certification scheme: Project based

[D] All of the above

Q.68) Which among the following is not one of the mandatory components of the National AYUSH
Mission (NAM)?
[A] AYUSH Services

[B] Medicinal Plants

[C] AYUSH Educational Institutions

[D] Tele-medicine

Q.69) Union government had approved continuation of the National Ayush Mission as a Centrally
Sponsored Scheme till the year _______.

[A] 2024

[B] 2023

[C] 2026

[D] 2025

Q.70) With respect to the 'National Green Hydrogen Mission', which of the following options is
incorrect in terms of the distribution of financial outlay of the scheme?

[A] The financial outlay for SIGHT Programme is Rs. 17,490 Crore

[B] The financial outlay for Pilot Projects is Rs. 1,466 Crore

[C] The financial outlay for R & D is Rs. 500 Crore

[D] The financial outlay other Mission Components is Rs. 388 Crore.

Q.71) Which of the following statements is/are the target/s to be achieved by 2030 under 'National
Green Hydrogen Mission'?

A. Development of Green Hydrogen Production capacity of at least 5 MMT per annum


https://edutap.learnyst.com/admin/products/all-questions/385022 18/71
3/29/23, 10:06 AM SchemesTap - January 2023 - Quiz-complete-test

B. Cumulative reduction in fossil fuel imports over Rs. 2 lakh crore

C. Abatement of nearly 50 MMT of annual greenhouse gas emissions


[A] Only A
[B] Only B

[C] Both A and B

[D] Both A and C

Q.72) Which of the following statements is/are correct with respect to the 'National Green Hydrogen
Mission'?

[A] Under the SIGHT Programme, 3 distinct financial incentive programmes will be provided under the
scheme

[B] The Mission will also support pilot projects in emerging end-use sectors and production pathways

[C] A public-private partnership framework for R&D (Strategic Hydrogen Innovation Partnership – SHIP)
will be facilitated under the Mission

[D] Only B and C

Q.73) Which of the following is not one of the goals of the Green India Mission?

[A] Increase the income of tree based products

[B] Increased forest/ tree cover

[C] Increased forest-based livelihood income of households living in and around forests.

[D] Improved quality of forest cover in non-forest lands

Q.74) In which of the following areas does the Green India Mission (GIM) focuses on?

A. Biodiversity

B. Water

C. Biomass

D. Mangroves

E. Wetlands 
[A] A, B, C and D

[B] A, C, D and E

[C] B, C, D and E

[D] A, B, C, D and E

https://edutap.learnyst.com/admin/products/all-questions/385022 19/71
3/29/23, 10:06 AM SchemesTap - January 2023 - Quiz-complete-test

Q.75) Which of the following statements is/are not correct with reference to the Green India Mission?

A. The Mission would strive for enhancing carbon sinks in sustainably managed forests.

B. Traditional Ecological Knowledge of communities, along with forestry science and state-of-the-art
technology would improve the Mission interventions. 

C. It aims at convergence with complementary schemes and programmes for better coordination in
developing forests and their fringe areas in a holistic and sustainable way.

[A] A only

[B] B only

[C] C only
[D] None of the above

Q.76) The National Programme for Dairy Development is to be implemented till ______.

[A] 2022-23

[B] 2023-24

[C] 2024-25
[D] 2025-26

Q.77) Which of the following is not one of the objectives of the National Programme for Dairy
Development?

[A] To create and strengthen infrastructure for production of quality milk.

[B] To create training infrastructure for training of dairy farmers.


[C] To strengthen dairy cooperative societies/producers companies at village level.

[D] All of the above

Q.78) What is the budget outlay of the National Dairy Development Programme?

[A] Rs 1220 crore

[B] Rs 1790 crore

[C] Rs 2110 crore


[D] Rs 2440 crore

Q.79) Which of the following statements is/are correct with reference to the Nirbhaya Fund?

https://edutap.learnyst.com/admin/products/all-questions/385022 20/71
3/29/23, 10:06 AM SchemesTap - January 2023 - Quiz-complete-test

A. The Nirbhaya Fund Framework provides for a non-lapsable corpus fund for safety and security of
women.

B. It is administered by the  Department of Economic Affairs of the Ministry of Finance of the


Government of India.

C. It is a dedicated fund which can be utilized for projects specifically designed to improve the safety
and security of women.
[A] A and B only

[B] A and C only

[C] B and C only


[D] A, B and C

Q.80) Under the Nirbhaya Fund, one of the schemes namely One Stop Centre Scheme is implemented
across the country since 1st April _____.

[A] 2013

[B] 2014
[C] 2015

[D] 2016

Q.81) What is the Women help Line number under the Nirbhaya Fund?

[A] 191

[B] 181
[C] 199

[D] 198

Q.82) Which of the following statements is/are correct with respect to the North East Special
Infrastructure Development Scheme (NESIDS)?

A. The funds are released in two instalments of 60% and 40%.

B. It is a Central Sector Scheme.


C. The NESIDS will be over and above the existing schemes of Government of India and State
Governments of the NE Region.

[A] A and B

[B] A and C

[C] B and C

https://edutap.learnyst.com/admin/products/all-questions/385022 21/71
3/29/23, 10:06 AM SchemesTap - January 2023 - Quiz-complete-test

[D] A, B and C

Q.83) In which year was the North East Special Infrastructure Development Scheme (NESIDS)
launched?
[A] 2015

[B] 2017

[C] 2018

[D] 2019

Q.84) What is the amount that will be released under the first instalment of the North East Special
Infrastructure Development Scheme (NESIDS)?
[A] Rs. 5 lakh

[B] Rs. 10 lakh

[C] Rs. 20 lakh

[D] Rs. 40 lakh

Q.85) Which of the following statement(s) is/are correct with respect to the One Stop Centre Scheme?
A. This scheme provides support all women including girls below 18 years.

B. The scheme will be funded through the ‘Nirbhaya Fund’.

C. Funds would be made available by Ministry of Women and Child Development to the State Women
Welfare departments.

[A] A and B
[B] B and C

[C] A and C

[D] A, B and C

Q.86) Which of the following is/are the location preference for setting up of One Stop Centres?

A. Suitable and adequate accommodation with separate access having at least 5 rooms and carpet area
of 132 sq.m. within a hospital / medical facility.
B. Existing Govt/Semi Govt institutions/Women Institutions/Swadhar Grehs/Working Women Hostels
located within 2 km radius of the hospital/medical facility

C. Centre may be constructed on adequate land either within hospital/medical facility or within 2 km
radius, having at least an area of 300 sq.m.
https://edutap.learnyst.com/admin/products/all-questions/385022 22/71
3/29/23, 10:06 AM SchemesTap - January 2023 - Quiz-complete-test

[A] A and B

[B] B and C

[C] A and C
[D] A, B and C

Q.87) Which of the following is not among the services provided by One Stop Centres?

[A] Medical assistance (Referral to hospital through helpline/Centre/Ambulance facility)

[B] Police assistance (Assisting in filing First Information Report)

[C] Psycho-social support/ counselling (Empanelled counsellors – pro-bono basis or for honorarium)

[D] Assistance in self-employment through interest free loans.

Q.88) What is the funding pattern (centre : state) under the Paramparagat Krishi Vikas Yojana (PKVY)
for non-Himalayan states?

[A] 50:50

[B] 60:40

[C] 70:30
[D] 40:60

Q.89) Which of the following statements is/are correct with respect to the Paramparagat Krishi Vikas
Yojana (PKVY)?

A. It promotes commercial organic production through certified organic farming.

B. It aims at forming clusters each of 50 Acre land to take up the organic farming.
C. Financial assistance of Rs 50000/ha/3 years is provided for cluster formation, capacity building,
incentive for inputs, value addition and marketing.

[A] A and B

[B] B and C

[C] A and C

[D] A, B and C

Q.90) Which of the following is/are correct with respect to the financial assistance provided under
Paramparagat Krishi Vikas Yojana (PKVY)?

A. Rs 31000/ha/3 years for preparation/procurement of organic inputs such as bio/organic fertilisers,


biopesticides, seeds etc
https://edutap.learnyst.com/admin/products/all-questions/385022 23/71
3/29/23, 10:06 AM SchemesTap - January 2023 - Quiz-complete-test

B. Rs 8800/ha/3 years for value addition and marketing that includes post-harvest management
practices like storage

C. Rs 3000 /hectare for 3 years for Cluster formation (of 20 ha) and Capacity building including
exposure visits, and trainings of field functionaries
[A] A and C

[B] B and C

[C] A and B

[D] A, B and C

Q.91) Which of the following statements is/are correct with respect to the PLI Scheme for
Pharmaceuticals?

A. It is being implemented by Ministry of Chemicals and Fertilizers.

B. It was launched in the year 2021.

C. The applicants eligible for the above schemes have been categorized into 4 groups based on Global
Manufacturing Revenue (FY 2019-20) of pharmaceutical goods.
[A] A and B

[B] A and C

[C] B and C

[D] A, B and C

Q.92) What is the total budgetary outlay for the PLI Scheme for Pharmaceuticals?
[A] Rs. 10000 Crore

[B] Rs. 12500 Crore

[C] Rs. 15000 Crore

[D] Rs. 20000 Crore

Q.93) Which among the following statements are correct with respect to the PLI Scheme for
Pharmaceuticals?
A. This scheme will be implemented from FY 2020-21 to 2028-29.

B. The eligible products have been categorized into three categories.

C. Rate of incentive will be 10% (of incremental sales value) for Category 1 and Category 2 products for
first four years and 5% (of incremental sales value) for Category 3 products for first four years.

[A] A and B

https://edutap.learnyst.com/admin/products/all-questions/385022 24/71
3/29/23, 10:06 AM SchemesTap - January 2023 - Quiz-complete-test

[B] A and C

[C] B and C

[D] A, B and C

Q.94) Which of the following statements is/are correct with respect to the Production Linked Incentive
(PLI) Scheme for Promotion of Domestic Manufacturing of Medical Devices?

A. It is being implemented by Ministry of Chemicals and Fertilizers.

B. It was launched in the year 2021.

C. A maximum of 28 applicants shall be selected under the Scheme basis score obtained as per the
Evaluation.
[A] A and B

[B] A and C

[C] B and C

[D] A, B and C

Q.95) What is the total budgetary outlay for the Production Linked Incentive (PLI) Scheme for
Promotion of Domestic Manufacturing of Medical Devices?

[A] Rs. 1750 Crore

[B] Rs. 2250 Crore

[C] Rs. 3420 Crore

[D] Rs. 4550 Crore

Q.96) Which among the following statements are correct with respect to the Production Linked
Incentive (PLI) Scheme for Promotion of Domestic Manufacturing of Medical Devices?

A. The incentives under the scheme will be implemented from FY 2021-22 to FY 2026-27.

B. 6% incentive will be provided on incremental sales (over Base Year: FY 2019-20) of medical devices
manufactured in India.

C. It will lead to expected incremental production of Rs. 78,437 crore over a period of five years.
[A] A and B

[B] A and C

[C] B and C

[D] None of the above

https://edutap.learnyst.com/admin/products/all-questions/385022 25/71
3/29/23, 10:06 AM SchemesTap - January 2023 - Quiz-complete-test

Q.97) Which of the following statements is/are correct with respect to the Pradhan Mantri Awaas
Yojana - Gramin?

A. It is being implemented by Ministry of Rural Development.


B. It aims at constructing 3.95 crore PMAY-G houses with all basic amenities by the year 2024.

C. The grants under the scheme are shared between the Centre and States in the ratio of 90:10 in case
of NE States, Himalayan States & Himalayan UTs.

[A] A and B

[B] A and C

[C] B and C
[D] A, B and C

Q.98) What is the minimum size of the house under the 'Pradhan Mantri Awaas Yojana - Gramin'?

[A] 20 sq.mt

[B] 25 sq.mt

[C] 30 sq.mt
[D] 35 sq.mt

Q.99) What is the unit assistance under the Pradhan Mantri Awaas Yojana - Gramin for hilly regions?

[A] Rs 0.70 lakh

[B] Rs 1.00 lakh

[C] Rs 1.20 lakh


[D] Rs 1.30 lakh

Q.100) Which of the following statements is/are correct with respect to thePM Gati Shakti - National
Master Plan?

A. Gati Shakti Digital Platform will bring 16 Ministries and 7 core infrastructure sectors together.

B. It was launched in the year 2020.

C. The scheme will be implemented for a duration of 4 years.


[A] A and B

[B] A and C

[C] B and C

[D] A, B and C

https://edutap.learnyst.com/admin/products/all-questions/385022 26/71
3/29/23, 10:06 AM SchemesTap - January 2023 - Quiz-complete-test

Q.101) Which of the following statements is/are correct with respect to the PM Gati Shakti - National
Master Plan?

A. It plans to establish 21 industrial corridors.


B. A GIS based ERP system, will be developed in collaboration with BISAG-N.

C. The Empowered Group of Secretaries (EGOS) has been mandated to review and monitor
implementation of the PM GatiShakti NMP to ensure logistics efficiency.

[A] A and B

[B] A and C

[C] B and C
[D] A, B and C

Q.102) Which among the following is not one of the pillars of 'PM Gati Shakti - National Master Plan'?

[A] Dynamic

[B] Analytical

[C] Comprehensiveness
[D] Responsiveness

Q.103) Which of the following statements is/are incorrect with respect to Pradhan Mantri Matru
Vandana Yojana (PMMVY)?

A. This scheme has been included as a sub-scheme under Mission Shakti.

B. Financial assistance of Rs. 5,000 are provided in THREE instalments.


C. This maternity benefit programme is implemented in accordance with the provision of the National
Food Security Act, 2013.

[A] A and B

[B] B and C

[C] A and C

[D] Only B

Q.104) Who among the following is/are not eligible under the 'Pradhan Mantri Matru Vandana Yojana
(PMMVY)'?

A. Geeta Devi, an ASHA worker in second trimester of pregnancy

B. Bhuri Bai, a tribal woman expecting her first child


https://edutap.learnyst.com/admin/products/all-questions/385022 27/71
3/29/23, 10:06 AM SchemesTap - January 2023 - Quiz-complete-test

C. Reena Malhotra, working as HR manager at ONGC Ltd.


[A] Only A

[B] Only B
[C] Only C

[D] A and C

Q.105) Which of the following statement is incorrect with respect to the 'Pradhan Mantri Matru
Vandana Yojana (PMMVY)'?

[A] This scheme is being implemented by Ministry of Women and Child Development.

[B] The case of miscarriage shall be considered only once.


[C] In case of infant mortality, benefits cannot be claimed if the woman has already received all the
instalments earlier.

[D] None of the above

Q.106) Pradhan Mantri Virasat Ka Samvardhan aims to empower which of the following communities?

[A] Youth
[B] Women

[C] Artisan communities

[D] All 1, 2 and 3

Q.107) Pradhan Mantri Virasat Ka Samvardhan provides how many hours of intensive training to
women?
[A] 200

[B] 100

[C] 300

[D] 400

Q.108) Pradhan Mantri Virasat Ka Samvardhan provides _____ days training for business mentors.

[A] 30
[B] 15

[C] 45

[D] 60

https://edutap.learnyst.com/admin/products/all-questions/385022 28/71
3/29/23, 10:06 AM SchemesTap - January 2023 - Quiz-complete-test

Q.109) Which of the following statements is/are correct with respect to the Rashtriya Gokul Mission
(RGM)?

A. It is being implemented by Department of Animal Husbandry and Dairying.


B. It was launched in the year 2015.

C. Rashtriya Gokul Mission (RGM) has been continued for 2021-22 to 2025-26.

[A] A and B

[B] A and C

[C] B and C

[D] A, B and C

Q.110) The components under the Rashtriya Gokul Mission (RGM) provides for 100% grant in aid
except for which among the following scenarios?

A. Rs 5000 per IVF pregnancy will be made available to participating farmers as GoI share.

B. Upto 50% of the cost of sex sorted semen will be made available to participating farmers.

C. Upto 50% of the capital cost maximum upto Rs.2.00 crore of the project will be made available to
entrepreneur for establishment of breed multiplication farm.

[A] A and B

[B] A and C

[C] B and C

[D] A, B and C

Q.111) Which among the following is not one of the components under the 'Rashtriya Gokul Mission
(RGM)'?

[A] Availability of High genetic Merit Germplasm

[B] Extension of Artificial Insemination Network

[C] Development and Conservation of exotic Breeds

[D] Skill Development

Q.112) Which of the following statements is/are correct with respect to the 'Rashtriya Gram Swaraj
Abhiyan (RGSA)'?

A. It is being implemented by Ministry of Panchayat Raj.

B. It was launched in the year 2018.

https://edutap.learnyst.com/admin/products/all-questions/385022 29/71
3/29/23, 10:06 AM SchemesTap - January 2023 - Quiz-complete-test

C. It is a centrally sponsored scheme.


[A] A and B

[B] A and C
[C] B and C

[D] A, B and C

Q.113) What is the total outlay of 'Rashtriya Gram Swaraj Abhiyan (RGSA)'?

[A] Rs. 2755.50 Crore

[B] Rs. 3500 Crore

[C] Rs. 4500 Crore


[D] Rs. 5911 Crore

Q.114) What is the number of aspirational districts to which the 'Rashtriya Gram Swaraj Abhiyan
(RGSA)' has been extended?

[A] 99

[B] 117
[C] 110

[D] 115

Q.115) Which of the following statements is/are correct with respect to the 'Rashtriya Krishi Vikas
Yojana (RKVY)'?

A. It is being implemented by Ministry of Agriculture and Farmers Welfare.


B. It was launched in the year 2007.

C. Till 2013-14, the scheme was implemented as an Additional Central Assistance (ACA) to State Plan
Scheme with 100% central assistance.

[A] A and B

[B] A and C

[C] B and C
[D] A, B and C

Q.116) Which among the following sectors are covered under the allied sectors of the 'Rashtriya Krishi
Vikas Yojana (RKVY)'?

[A] Animal Husbandry, Dairy Development and Fisheries


https://edutap.learnyst.com/admin/products/all-questions/385022 30/71
3/29/23, 10:06 AM SchemesTap - January 2023 - Quiz-complete-test

[B] Agricultural Marketing

[C] Agricultural Financial Institutions

[D] All of the above

Q.117) Which among the following is not one of the sub-schemes under the 'Rashtriya Krishi Vikas
Yojana (RKVY)'?

[A] Crop Diversification Program

[B] Reclamation of Problem Soil

[C] Bringing Green Revolution to Northern Eastern India

[D] National Saffron Mission

Q.118) Which of the following persons will get assistance from the ADIP scheme (Assistance to
Disabled Persons for Purchase / Fitting of Aids and Appliances)?

1. Joseph who is a Indian citizen of age 72.

2. Alex who has 50% disability.

3. Maria who is deaf and her income is above 30000.


[A] A only

[B] B and C only

[C] A and B only

[D] C only

Q.119) Which of the following statements are correct regarding the financial assistance provided under
the ADIP scheme (Assistance to Disabled Persons for Purchase / Fitting of Aids and Appliances)?

A. Full financial assistance for aids/appliances costing upto Rs. 15,000.

B. Financial assistance upto Rs. 20000 for appliances costing between Rs. 15,001 to Rs. 30000.

C. Government of India shall bear 50% of cost of items  costing above Rs. 30,001/, except Cochlear
Implant and Motorized Tricycle.

[A] A and B only


[B] A and C only

[C] C and A only

[D] A, B and C

https://edutap.learnyst.com/admin/products/all-questions/385022 31/71
3/29/23, 10:06 AM SchemesTap - January 2023 - Quiz-complete-test

Q.120) Which of the following are the objectives of Shahari Waqf Sampatti Vikas Yojana?

A. To improve the financial position of the auqaf

B. To obtain occupied Waqf land from encroachers


C. To develop economically viable projects on the properties to enable income

[A] A only

[B] B only

[C] C only

[D] A and C only

Q.121) What is the maximum percentage of the estimated cost of the project can be obtained as
assistance under the Shahari Waqf Sampatti Vikas Yojana?

[A] 60%

[B] 75%

[C] 80%

[D] 85%

Q.122) The loan obtained by the Waqf boards under the Shahari Waqf Sampatti Vikas Yojana must be
generally paid in which of the following manner?

[A] 16 half yearly instalments

[B] 8 annual instalments

[C] 32 quarterly instalments


[D] 96 monthly instalments

Q.123) Which of the following statements is/are correct with respect to the 'Skill India Mission'?

A. It is being implemented by Ministry of Skill Development and Entrepreneurship.

B. It was launched in the year 2015.

C. It aims to provide market-relevant skills training to more than 50 crore young people in the country
by the year 2022.
[A] A and B

[B] A and C

[C] B and C

[D] A, B and C

https://edutap.learnyst.com/admin/products/all-questions/385022 32/71
3/29/23, 10:06 AM SchemesTap - January 2023 - Quiz-complete-test

Q.124) Which of the following statements is/are correct with respect to the 'Skill India Mission'?

A. It offers short term and long term training.

B. The mission offers training, guidance, and support for all traditional types of employment.
C. Training imparted would be of international standards.

[A] A and B

[B] A and C

[C] B and C

[D] A, B and C

Q.125) Which among the following is not one of the sub-schemes under the 'Skill India Mission'?
[A] National Skill Development Mission (NSDM)

[B] Indian Skill Development Service

[C] Skill Loan Scheme

[D] All of the above

Q.126) Which of the following schemes aims to strengthen the existing infrastructure facilities in order
to make India a global leader in the Pharma Sector?

[A] Janaushadhi Suvidha Oxy-Biodegradable Sanitary Napkin Scheme

[B] Production Linked Incentive (PLI) Scheme for Bulk Drugs

[C] Production Linked Incentive Schemes of Pharmaceuticals

[D] Strengthening of Pharmaceutical Industry (SPI)' Scheme

Q.127) An outlay of  how many crore rupees has been earmarked for the Pharmaceutical & Medical
Devices Promotion and Development Scheme for period of five years?

[A] 30.5

[B] 17.5

[C] 21.5

[D] 43.5

Q.128) What is the credit linked capital subsidy provided under the Pharmaceutical Technology
Upgradation Assistance Scheme?

https://edutap.learnyst.com/admin/products/all-questions/385022 33/71
3/29/23, 10:06 AM SchemesTap - January 2023 - Quiz-complete-test

[A] 8%

[B] 9%

[C] 10%
[D] 11%

Q.129) Which of the following statements is incorrect with respect to the ‘Swachh Bharat Mission –
Grameen’?

A. Under SBM(G) Phase 2, Gobardhan initiative has been launched to convert bio-waste into biogas and
bio slurry.

B. Under SBM(G) Phase-II, financial assistance of up to Rs. 50 lakh per district is available for setting up
of community and cluster model biogas plants.

C. Geo-tagging of toilets has been made mandatory under SBM(G).

[A] A and B

[B] B and C

[C] A and C
[D] None of the above

Q.130) Which of the following statements is/are correct with respect to the SBM(G)?

A. It is operated as a Central Sector Scheme

B. Tenure - From 2020-21 till 2024-25

C. Financial outlay for this scheme is Rs. 1,40,881 crores


[A] A and C

[B] B and C

[C] A and B

[D] A, B and C

Q.131) How much amount has been allocated in the budget for the year 2022-23 for Swachh Bharat
Mission - Grameen?
[A] Rs 7,011 Crore 

[B] Rs 7,192 Crore 

[C] Rs 7,541 Crore 

[D] Rs 7,856 Crore 

https://edutap.learnyst.com/admin/products/all-questions/385022 34/71
3/29/23, 10:06 AM SchemesTap - January 2023 - Quiz-complete-test

Q.132) Which of the following is/are the objectives of Swachh Bharat Mission (Grameen) Phase II?

A. Open defecation free behaviours are sustained and no one is left behind.

B. To encourage cost effective and appropriate technologies for ecologically safe and sustainable
sanitation.

C. To create significant positive impact on gender and promote social inclusion by improving sanitation
especially in marginalized communities.

[A] A and B only       

[B] B and C only

[C] A and C only


[D] A, B and C

Q.133) What is the maximum interest subvention provided by the central government under the
Weaver Mudra scheme?

[A] 4%

[B] 5%
[C] 6%

[D] 7%

Q.134) Under the Weaver Mudra Scheme, the margin money assistance is provided for ____% of loan
amount, subject to maximum of Rs. ______ per weaver.

[A] 10, 20000


[B] 20, 25000

[C] 30, 40000

[D] 40, 55000

Q.135) The Handloom Weaver MUDRA Portal has been developed in association with which of the
following banks?

[A] Punjab National Bank


[B] State Bank of India

[C] Indian Bank

[D] Reserve Bank of India

SchemesTap - January 2023 - Quiz


https://edutap.learnyst.com/admin/products/all-questions/385022 35/71
3/29/23, 10:06 AM SchemesTap - January 2023 - Quiz-complete-test

Answers
Government Schemes
Q.1)D Q.2)D Q.3)D Q.4)D Q.5)A Q.6)A Q.7)D Q.8)C Q.9)B Q.10)D Q.11)B Q.12)C

Q.13)D Q.14)D Q.15)C Q.16)A Q.17)D Q.18)B Q.19)C Q.20)D Q.21)D Q.22)D Q.23)D

Q.24)B Q.25)A Q.26)D Q.27)D Q.28)A Q.29)D Q.30)D Q.31)D Q.32)D Q.33)B Q.34)A

Q.35)A Q.36)D Q.37)B Q.38)D Q.39)B Q.40)B Q.41)A Q.42)A Q.43)B Q.44)D Q.45)A

Q.46)D Q.47)D Q.48)B Q.49)C Q.50)B Q.51)D Q.52)D Q.53)C Q.54)D Q.55)D Q.56)C

Q.57)D Q.58)D Q.59)B Q.60)D Q.61)A Q.62)C Q.63)D Q.64)D Q.65)A Q.66)B Q.67)D

Q.68)D Q.69)C Q.70)C Q.71)D Q.72)D Q.73)A Q.74)D Q.75)D Q.76)D Q.77)D Q.78)B

Q.79)D Q.80)C Q.81)B Q.82)C Q.83)B Q.84)B Q.85)A Q.86)D Q.87)D Q.88)B Q.89)D

Q.90)D Q.91)A Q.92)C Q.93)D Q.94)B Q.95)C Q.96)D Q.97)B Q.98)B Q.99)D

Q.100)B Q.101)C Q.102)D Q.103)D Q.104)C Q.105)D Q.106)D Q.107)B Q.108)A

Q.109)B Q.110)D Q.111)C Q.112)D Q.113)D Q.114)B Q.115)D Q.116)D Q.117)C

Q.118)C Q.119)B Q.120)D Q.121)B Q.122)A Q.123)A Q.124)D Q.125)D Q.126)D

Q.127)C Q.128)C Q.129)D Q.130)B Q.131)B Q.132)D Q.133)D Q.134)B Q.135)A

SchemesTap - January 2023 - Quiz


Explanations
Government Schemes
Q.1) Explanation:

·     Type of Scheme - Central Sector Scheme

·     Duration - Extended to 13 years up to 2032-33

·     Launch - 2020

Q.2) Explanation:

https://edutap.learnyst.com/admin/products/all-questions/385022 36/71
3/29/23, 10:06 AM SchemesTap - January 2023 - Quiz-complete-test

·     Rs. 1 Lakh Crore will be provided by banks and financial institutions as loans to eligible beneficiaries

·     Moratorium for repayment may vary subject to minimum 6 months and maximum of 2 years

·     Interest subvention of 3% per annum up to a limit of loans of Rs. 2 Crore

o   Subvention will be available for a maximum period of 7 years

Q.3) Explanation:
Beneficiaries:

·     Farmers, Farmer Producer Organizations, Primary Agricultural Credit Society

·     Marketing/Multipurpose Co-operative Societies, SHGs, Joint Liability Groups

·     Agri-entrepreneurs, Start-ups

·     Central/State agency or Local Body sponsored PPP Projects

·        State agencies and Agricultural Produce Marketing Committees (APMCs)

Q.4) Explanation:

·     AMRUT 2.0 is to be implemented for a tenure of 2021-22 till 2025-26.

Q.5) Explanation:

·     The total indicative outlay for AMRUT 2.0 is ₹2,99,000 crore
 

Q.6) Explanation:

·     The total indicative outlay for AMRUT 2.0 is ₹ 2,99,000 crore including central share of ₹ 76,760
crore for five years from FY 2021-22 to FY 2025-26.

·     Central Government share in projects -


o   For UTs: 100% central share

o   North-Eastern & Himalayan States: 90% central share (Statement B is incorrect)

o   Population less than 1 lakh: 50% of project funds by center

o   Population between 1 to 10 lakh: One-third of the project cost by center

o   Population more than 10 lakh: 25% of project cost by center

https://edutap.learnyst.com/admin/products/all-questions/385022 37/71
3/29/23, 10:06 AM SchemesTap - January 2023 - Quiz-complete-test

·     It is mandated for cities having million plus population to take up PPP projects worth minimum of
10% of their total project fund allocation which could be on Annuity/ Hybrid Annuity / BOT Model
(Statement C is incorrect).

Q.7) Explanation:
·     It is a centre sector scheme. (Hence, statement 1 is incorrect)

·       It has been set up with Rs.15,000 Crores under AtmaNirbhar Bharat Abhiyaan Stimulus Package.
(Hence, statement 2 is incorrect)

·        The Central Government is providing credit guarantee of 25% of total borrowing for MSME projects.
(Hence, statement 3 is incorrect)

Q.8) Explanation:
 

·     The government will provide 3% interest subvention.

Q.9) Explanation:
·     2 years moratorium period for principal amount & 6 years repayment period will be provided. (Hence,
statement 1 is incorrect)

·     Credit Guarantee Fund of Rs. 750 Crores is set up by government and managed by NABARD. (Hence,
statement 2 is correct)

·              The Nodal Ministry is Ministry of Fisheries, Animal Husbandry and Dairying. The implementing
agency is Animal Husbandry and Dairying. (Hence, statement 3 is incorrect)

·        Individual entrepreneurs are included under the scheme. (Hence, statement 4 is incorrect)

Q.10) Explanation:

·    Atal Bhujal Yojana (Atal Jal) is implemented by the Ministry of Jal Shakti (Statement A is incorrect).

·    It was launched in the year 2019 (Statement B is incorrect).

https://edutap.learnyst.com/admin/products/all-questions/385022 38/71
3/29/23, 10:06 AM SchemesTap - January 2023 - Quiz-complete-test

·       This scheme implemented for 5 years from 2020-21 to 2024-25 (Statement C is incorrect).

Q.11) Explanation:
·    The total outlay of the scheme is Rs. 6000 Crore (Option 5 is correct).

·    Rs. 3000 Crore will be taken as loan from World Bank.

Q.12) Explanation:
·    It is Central Sector Scheme (Statement A is incorrect).

·    The scheme also aims at bringing about behavioral change at the community level through awareness
programs and capacity building for fostering sustainable ground water management (Statement B is correct).

·            The scheme has 2 components, incentive component and institutional strengthening and capacity
building component (Statement C is correct).

Q.13) Explanation:
Atal Bhujal Yojana is being funded by World Bank and Government of India on 50:50 basis (Option 4
is correct).

Q.14) Explanation:

·     Objectives
o    To promote afforestation and development activities in order to compensate for forest land that is
intended to be diverted to non-forest uses.

o   To law down effective guidelines for the State.

o    To facilitate necessary assistance in terms of scientific, technological and other requisites that may be
required by the authority responsible for the State CAMPA.

o   To recommend measures based on strategic planning to the authorities of the State CAMPA.

o   To resolve issues that arise between inter-state or Centre-State.

https://edutap.learnyst.com/admin/products/all-questions/385022 39/71
3/29/23, 10:06 AM SchemesTap - January 2023 - Quiz-complete-test

Q.15) Explanation:

·     CAMPS is operated under the chairmanship of the Union Minister of Environment & Forests

Q.16) Explanation:
·       90% of the funds collected will be transferred to State CAMPA and remaining 10% will be with
National CAMPA for utilization.

Q.17) Explanation:

·     Eligible Activity – Manufacturing and services including Retail Trade allowed. Educational / Training
Institutions, SHG and agriculture are ineligible for coverage.

·     The corpus of CGTMSE is being contributed by the GoI and SIDBI in the ratio of 4:1 respectively

·        The Ministry of MSMEs, GoI and Small Industries Development Bank of India (SIDBI) established a
trust named  Credit Guarantee Fund Trust for Micro and Small Enterprises (CGTMSE)  to implement the
CGS.

·        Fund and non-fund based (Letters of Credit, Bank Guarantee etc.) credit facilities up to Rs 200 lakh
per eligible borrower are covered under the guarantee scheme provided they are extended on the project
viability without collateral security or third party guarantee.

Q.18) Explanation:
·     Credit Guarantee Fund Trust for Micro and Small Enterprises (CGTMSE) Scheme was launched in the
year 2000.

Q.19) Explanation:
·     The extent of guarantee cover is 85% for micro enterprises for credit up to Rs 5 lakh.

·     The extent of guarantee cover is 50% of the sanctioned amount of the credit facility for credit from Rs
10 lakh to Rs 100 lakh per MSE borrower for retail trade activity.

·        The extent of guarantee cover is 80%.

o   Micro and Small Enterprises operated and/or owned by women

o   All credits/loans in the North East Region (NER) for credit facilities upto Rs 50 lakh

https://edutap.learnyst.com/admin/products/all-questions/385022 40/71
3/29/23, 10:06 AM SchemesTap - January 2023 - Quiz-complete-test

·        In case of default, Trust settles the claim up to 75% of the amount in default of the credit facility
extended by the lending institution for credit facilities up to Rs 200 lakh.

Q.20) Explanation:

·       Digital Health Incentive Scheme (DHIS) has been launched by National Health Authority (NHA).
(Statement A correct).

·     It was launched in the year 2022. (Statement B is correct).

·     It aims to give a further boost to digital health transactions in the country under the Ayushman
Bharat Digital Mission (ABDM). (Statement C is correct).

Q.21) Explanation:
·       The scheme offers incentives of up to Rs. 4 crores to hospitals, diagnostic labs and digital health
solution providers such as Hospital/ Health Management Information System (HMIS) and Laboratory
Management Information System (LMIS).

Q.22) Explanation:
·     Under the scheme, incentives would be provided to the following entities-

o   Health Facilities having 10 or more beds

o   Laboratory/radiology diagnostics centres

o   Digital Solution Companies (entities providing ABDM enabled digital solutions)

Q.23) Explanation:
The EEAT scheme comprises of three programmes -

·     National Green Corps “Ecoclub” programme

·     National Nature Camping programme

·        Capacity Building Activities

Q.24) Explanation:

https://edutap.learnyst.com/admin/products/all-questions/385022 41/71
3/29/23, 10:06 AM SchemesTap - January 2023 - Quiz-complete-test

·     Environment Education, Awareness and Training (EEAT) Scheme was launched in the year 1983-84.

Q.25) Explanation:

·       Fast Track Special Court Scheme is implemented by the Department of Justice (Statement A is
correct).
·     The financing of the Scheme will be on the pattern of Centrally Sponsored Scheme.

o   Central Share is to be funded from Nirbhaya Fund.

·       Setting up Fast Track Courts (FTCs) and its functioning comes within the domain of the State
Governments who set up such courts in consultation with respective High Courts. 

Q.26) Explanation:
·     Fast Track Special Courts Scheme was launched in the year 2019.

Q.27) Explanation:

·     Department of Justice is implementing a centrally sponsored scheme for setting up of 1023 Fast
Track Special Courts (FTSCs) including 389 exclusive courts for expeditious trial and disposal of cases
related to rape and POCSO Act.

Q.28) Explanation:
FIDF entails an estimated fund size of Rs.7,522 crore, comprising Rs.5,266.40 crore to be raised by
ASthe Nodal Loaning Entities (NLEs), Rs. 1,316.6 crore beneficiary’s contribution and Rs.939.48 crore
budgetary support from the Government of India.

Q.29) Explanation:
·     It is a Central Sector Scheme.

·     National level de-duplication of all ration cards/beneficiaries’ data.

·     Integration of States/UTs PDS systems/applications with Central PDS systems/applications.

·     Use of advanced data analytics techniques to bring about continuous improvements in PDS operations.

·        Development of advanced web and mobile-based applications.

·        Facilitation of cross-learning and sharing of best practices between States/UTs, etc.
https://edutap.learnyst.com/admin/products/all-questions/385022 42/71
3/29/23, 10:06 AM SchemesTap - January 2023 - Quiz-complete-test

Q.30) Explanation:
·     Integrated Management of Public Distribution System (IMPDS) Scheme was launched in the year 2018.

Q.31) Explanation:

·     Ministry of Social Justice & Empowerment is the nodal ministry for the Integrated Programme for
Senior Citizens (IPSrC) (Statement A correct).

·       Department of Social Justice and Empowerment is the implementing authority of the scheme
(Statement B is correct).
·        It is a central sector scheme. (Statement C is correct).

Q.32) Explanation:
·     Under the above mentioned scheme, upto 90% of the cost of the project is provided by Government of
India, while remaining shall be borne by Organization/ Institution concerned.

Q.33) Explanation:

·     Theme - “Catch the rain, where it falls, when it falls”.

·     It covers all rural and urban areas of all districts of the country.

o   Jal Shakti Abhiyan-I of 2019, covered only 1592 water stressed blocks out of 2836 blocks in 256
districts of the country.

·        Creation of new and maintenance of old Rainwater Harvesting Structures, revival of traditional
rainwater harvesting structures like step wells etc, enumeration, geo-tagging and making inventory of
all water bodies, preparation of scientific water conservation plans, setting up of Jal Shakti Kendras and
intensive afforestation are part of the campaign.

Q.34) Explanation:
·     Jal Shakti Abhiyan:Catch the Rain Scheme was launched in the year 2021.

 
https://edutap.learnyst.com/admin/products/all-questions/385022 43/71
3/29/23, 10:06 AM SchemesTap - January 2023 - Quiz-complete-test

Q.35) Explanation:

·     Nodal Agency: National Water Mission, under the Ministry of Jal Shakti

Q.36) Explanation:

·     It is funded through 100% annual grant from the Government of India

·     It was transferred from Ministry of Human Resource Development to Ministry of Skill Development
and Entrepreneurship in July, 2018

·        This scheme is implemented through NGOs.

Q.37) Explanation:
New Guidelines released by Central Government in January 2019

·       Alignment of JSS course and curriculum with National Skill Qualification Framework (NSQF) to
provide standardize training across sectors

·       Decentralization of powers for JSSs by providing accountability and independence to district


administration.

·        To identify and promote traditional skills in the district through skilling / upskilling.

·        Evidence-based assessment system.

·        Easy Online certification.

·        Linking JSS to Public Finance Management system to maintain transparency and accountability of the
ecosystem.

·        Creating livelihood linkages.

·        Training of Trainers to develop the capacity through National Skills Training Institutes.

Q.38) Explanation:
Scope of Work of Jan Shikshan Sansthans

·     Develop appropriate curriculum and training modules covering vocational elements general awareness
and life enrichment components.

·       Training equivalent to courses designed by the Directorate of Adult education, National Institute of
Open Schooling and Director General Employment & Training.

https://edutap.learnyst.com/admin/products/all-questions/385022 44/71
3/29/23, 10:06 AM SchemesTap - January 2023 - Quiz-complete-test

·        Provide training to a pool of resource persons and master trainers for conducting training.

·        Administer simple tests and award certificates.

·        Network with employers and industries for trainees to get suitable placements.

Q.39) Explanation:

·     Make in India Scheme was launched in the year 2014 (Statement B is incorrect).

·       Department for Promotion of Industry and Internal trade (DPIIT) coordinates action plans for
manufacturing sectors while Department of Commerce coordinates service sectors (Statement A & C
are correct).
 

Q.40) Explanation:
·       The Make in India   initiative is based on four pillars, which have been identified to give boost to
entrepreneurship in India, not only in manufacturing but also other sectors:

o   New Processes: A number of initiatives have been undertaken to ease the business environment.

o   New Infrastructure: Develop industrial corridors and smart cities, create world class infrastructure with
state-of-the-art technology and high-speed communication.
o   New Sectors: FDI has been opened up in Defence Production, Insurance, Medical Devices, Construction
and Railway infrastructure in a big way. Similarly, FDI has been allowed in Insurance and Medical Devices.

o   New Mindset: In order to partner with industry in economic development of the country Government
shall act as a facilitator and not a regulator.

Q.41) Explanation:
·     Target

o   Target an increase in manufacturing sector growth to 12-14% per annum over the medium term.

o   An increase in the share of manufacturing in the country’s Gross Domestic Product from 16% to 25%
by 2022.

o   To create 100 million additional jobs by 2022 in manufacturing sector.

Q.42) Explanation:

https://edutap.learnyst.com/admin/products/all-questions/385022 45/71
3/29/23, 10:06 AM SchemesTap - January 2023 - Quiz-complete-test

·     The Scheme was launched in the year 2014.

·       Under MIDH, Government of India (GOI) contributes 60%, of total outlay for developmental
programmes in all the states except states in North East and Himalayas, 40% share is contributed by State
Governments.

·        In the case of North Eastern States and Himalayan States, GOI contributes 90%.

·        In case of National Horticulture Board (NHB), Coconut Development Board (CDB), Central Institute
for Horticulture (CIH), Nagaland and the National Level Agencies (NLA), GOI contributes 100%.

Q.43) Explanation:
·     It is a Centrally Sponsored Scheme.

·     This scheme is a sub-scheme of Green Revolution - Krishonnati Yojana.

·        There are 5 sub-schemes under MIDH –

o   National Horticulture Mission (NHM)

o   Horticulture Mission for North East and Himalayan States (HMNEH)

o   National Horticulture Board (NHB)

o   Coconut Development Board (CDB)

o   Central Institute of Horticulture (CIH), Nagaland.

Q.44) Explanation:
·     There are 5 schemes under MIDH –

o   National Horticulture Mission (NHM)

o   Horticulture Mission for North East and Himalayan States (HMNEH)

o   National Horticulture Board (NHB)

o   Coconut Development Board (CDB)

o   Central Institute of Horticulture (CIH), Nagaland.

Q.45) Explanation:

·       It aims to create at least one or two replicable end-to-end organic value chain models in every
State with the integration of handlers, growers, processors and market facilitation agencies.

https://edutap.learnyst.com/admin/products/all-questions/385022 46/71
3/29/23, 10:06 AM SchemesTap - January 2023 - Quiz-complete-test

·       It is a Central Sector Scheme, a sub-mission under National Mission for Sustainable Agriculture
(NMSA) – (Statement C is incorrect).

Q.46) Explanation:
·     The Scheme was launched in the year 2015.

Q.47) Explanation:
·       It is being implemented in the states of Arunachal Pradesh, Assam, Manipur, Meghalaya, Mizoram,
Nagaland, Sikkim and Tripura.

Q.48) Explanation:
·     Mission POSHAN 2.0 is implemented by the Ministry of Women and Child Development (Statement
A is correct).

·     Mission POSHAN 2.0 was launched in the year 2021 (Statement B is incorrect).

·        Mission POSHAN 2.0 is an Integrated Nutrition Support Programme (Statement C is correct).

Q.49) Explanation:
·     The total cost under Mission Poshan 2.0 has been calculated on the basis of cost sharing ratio between
the Central and States/UTs, as approved by Government i.e., 60:40 for States and UTs with legislature
(Statement A is incorrect), 90:10 for NER & Himalayan States and UT of J&K and 100% for Union
Territories without legislature.

Q.50) Explanation:
·     Mission POSHAN 2.0, is an umbrella scheme covering –

o   Integrated Child Development Services (ICDS)

o   Anganwadi Services

o   Poshan Abhiyaan

o   Scheme for Adolescent Girls


https://edutap.learnyst.com/admin/products/all-questions/385022 47/71
3/29/23, 10:06 AM SchemesTap - January 2023 - Quiz-complete-test

o   National Creche Scheme

Q.51) Explanation:
·       Mission Shakti is implemented by the Ministry of Women and Child Development (Statement A is
correct).

·     Mission Shakti has two sub-schemes ‘Sambal’ and ‘Samarthya’. (Statement B is correct)

o   While the “Sambal” sub-scheme is for safety and security of women, the “Samarthya” sub-scheme is for
empowerment of women.

·              This umbrella scheme has been approved for implementation during the period of 15th  Finance
Commission, i.e. from 2021-22 to 2025-26 with a total outlay of Rs 20,989 crores including Central Share of
Rs 15,761 crores and State Share of Rs 5228 crores (Statement C is correct).
 

Q.52) Explanation:
·     The Sambal sub-scheme consists of the existing scheme of One Stop Centres (OSC), Women Helplines
(181-WHL) and Beti Bachao Beti Padhao (BBBP).

Q.53) Explanation:
·       The “Samarthya'' sub scheme is for empowerment of women, consisting of existing schemes of
Ujjwala, Swadhar Greh and Working Women Hostel.

o   In addition, the National Creche Scheme for children of working mothers and the Pradhan Mantri Matru
VandanaYojana (PMMVY), which have been under the Umbrella ICDS Scheme till now, are also subsumed
in ‘Samarthya’

Q.54) Explanation:
·     The Scheme provides -

o    Capital Subsidy - 20% for investments in Special Economic Zones (SEZs) and 25% in non-SEZs
(Statement A is incorrect).

o   Incentives for both new units and expansion units (Statement B is incorrect).

o   Incentives for a period of 5 years from the date of approval of application (Statement C is incorrect).

o    Incentives for 44 categories/verticals across the value chain (raw materials including assembly,
testing, packaging and accessories, chips, components).

https://edutap.learnyst.com/admin/products/all-questions/385022 48/71
3/29/23, 10:06 AM SchemesTap - January 2023 - Quiz-complete-test

o    Minimum investment threshold for each product category/ vertical (from Rs 1 crore for
manufacturing of accessories to Rs 5000 crores for memory semiconductor wafer fabrication unit.

o   Unit to be in Industrial Area notified by Central/State Govt.

Q.55) Explanation:

·     Launch Year: 2012

Q.56) Explanation:
·     Museum Grant Scheme was launched in the year 2013.

Q.57) Explanation:
·     The maximum amount of financial assistance which may be given would be 80% of the total project
cost. 

o   In case of museums in North-Eastern region including Sikkim the financial assistance would be 90% of
the total project cost.
o   The remaining amount i.e. 20% of the project cost (in case of North Eastern region, 10% of the project
cost), will have to be borne by the organization.

Q.58) Explanation:
·     Components

o   Development and Establishment of Museums at the regional, state and District level

o   Digitization of Museum Collections

o   Capacity building and training of Museum Professionals

Q.59) Explanation:

·    The Pilot Scheme of Nagar Van Yojana (NVY) envisages developing 400 Nagar Vans and 200 Nagar
Vatikas in the country during the period of 2020-21 to 2024-25.

https://edutap.learnyst.com/admin/products/all-questions/385022 49/71
3/29/23, 10:06 AM SchemesTap - January 2023 - Quiz-complete-test

Objective: to significantly enhance the tree outside forests and green cover, enhancement of
biodiversity and ecological benefits to the urban and peri-urban areas apart from improving
quality of life of city dwellers.
 

Q.60) Explanation:
·     Nagar Van Scheme was launched in the year 2020.

Q.61) Explanation:

·     Warje Urban Forest in Pune (Maharashtra) will be considered as a role model for the Nagar Van
Scheme.

Q.62) Explanation:

Q.63) Explanation:

https://edutap.learnyst.com/admin/products/all-questions/385022 50/71
3/29/23, 10:06 AM SchemesTap - January 2023 - Quiz-complete-test

Q.64) Explanation:
Key Features

·     NAPS has replaced Apprentice Protsahan Yojana (APY). (Hence, statement A is incorrect.)

·     Courses approved by State Government/Central Government such as PMKVY, DDU-GKY etc. shall
be linked with apprenticeship training (Statement C is correct).

·              Online portal would be used for administering the implementation of the Apprenticeship
Training. It will address the requirements of all key stakeholders.

·        It covers all categories of apprentices except the Graduate, Technician and Technician (Vocational)
apprentices which are covered by the scheme administered by Ministry of Education (erstwhile
Ministry of Human Resource Development). (Hence, statement B is incorrect.)
 

Q.65) Explanation:
Components:

·              Share 25% of the prescribed stipend, subject to a maximum of Rs 1,500 per month per
apprentice to the employer.

·              Share of cost of basic training with Basic Training Providers (BTP); up to Rs 7,500 for 3
months/500 hours.

 
https://edutap.learnyst.com/admin/products/all-questions/385022 51/71
3/29/23, 10:06 AM SchemesTap - January 2023 - Quiz-complete-test

Q.66) Explanation:

·     National Ayush Mission is implemented by the Ministry of Ayush (Statement A is correct).

·     National Ayush Mission (NAM) was launched in the year 2014 (Statement B is incorrect).
·              The National AYUSH Mission intends to build on India’s unmatched heritage represented by its
ancient systems of medicine like Ayurveda, Sidhha, Unani & Homeopathy (ASU&H) which are a treasure
house of knowledge for preventive and promotive healthcare. (Statement C is correct).

Q.67) Explanation:
·     Flexible Component

o   AYUSH Wellness Centres comprising Yoga and Naturopathy,

o   Tele-medicine,

o   Innovations in AYUSH including Public Private Partnership,

o   IEC (Information, Education and Communication) activities,

o   Voluntary certification scheme: Project based, etc.

Q.68) Explanation:
·     Mandatory Components

o   AYUSH Services.

o   AYUSH Educational Institutions.

o   Quality Control of ASU&H (Ayurveda, Siddha and Unani & Homoeopathy) Drugs.

o   Medicinal Plants.

·     Flexible Component


o   AYUSH Wellness Centres comprising Yoga and Naturopathy,

o   Tele-medicine,

o   Innovations in AYUSH including Public Private Partnership,

o   IEC (Information, Education and Communication) activities,

o   Voluntary certification scheme: Project based, etc.

https://edutap.learnyst.com/admin/products/all-questions/385022 52/71
3/29/23, 10:06 AM SchemesTap - January 2023 - Quiz-complete-test

Q.69) Explanation:
·        In July 2021, Union Cabinet has approved continuation of the National Ayush Mission as a Centrally
Sponsored Scheme for another five years till 2026.

Q.70) Explanation:
·     The total Financial Outlay of National Green Hydrogen Mission is Rs.19,744 crore. It is further
categorised into:

-      Rs.17,490 crore for the SIGHT  programme

-        Rs.1,466 crore for pilot projects

-        Rs.400 crore for R&D

-        Rs. 388 crore towards other Mission components


 

Q.71) Explanation:
 

·    Targets of the Mission by 2030:

o    Development of green  hydrogen production capacity of at least 5 MMT (Million Metric Tonne) per
annum with an associated renewable energy capacity addition of about 125 GW in the country
o   Over Rs. Eight lakh crore in total investments

o   Creation of over Six lakh jobs

o   Cumulative reduction in fossil fuel imports over Rs. One lakh crore

o   Abatement of nearly 50 MMT of annual greenhouse gas emissions

Q.72) Explanation:
·    Key Features:

o   The Mission will facilitate demand creation, production, utilization and export of Green Hydrogen.

o      Under the Strategic Interventions for Green Hydrogen Transition Programme (SIGHT), two distinct
financial incentive mechanisms – targeting domestic manufacturing of electrolysers and production of Green
Hydrogen – will be provided under the Mission.
https://edutap.learnyst.com/admin/products/all-questions/385022 53/71
3/29/23, 10:06 AM SchemesTap - January 2023 - Quiz-complete-test

o    The Mission will also support pilot projects in emerging end-use sectors and production pathways. 

o   Regions capable of supporting large scale production and/or utilization of Hydrogen will be identified
and developed as Green Hydrogen Hubs.

o   A public-private partnership framework for R&D (Strategic Hydrogen Innovation Partnership – SHIP)
will be facilitated under the Mission; R&D projects will be goal-oriented, time bound, and suitably scaled up
to develop globally competitive technologies.

o   A coordinated skill development programme will also be undertaken under the Mission.

Q.73) Explanation:
Objectives of Green India Mission:

·        Increased forest/ tree cover

·        Improved quality of forest cover in millions of hectares of forest/ non-forest lands

·              Improved ecosystem services including biodiversity, carbon sequestration and hydrological


services along with provisioning services like fuel, fodder, and timber and non-timber forest produces

·        Increased forest-based livelihood income of households living in and around forests.

Q.74) Explanation:

·       GIM focuses on multiple ecosystem services such as biodiversity, water, biomass, preserving
mangroves, wetlands, critical habitats etc. along with carbon sequestration. 

Q.75) Explanation:

·     The Mission would strive for enhancing carbon sinks in sustainably managed forests and other
ecosystems, adaptation of vulnerable species/ ecosystems to the changing climate and adaptation of
forest-dependent communities.

·     Traditional Ecological Knowledge of communities, along with forestry science and state-of-the-art
technology would improve the Mission interventions. 

·              GIM also aims at convergence with complementary schemes and programmes for better
coordination in developing forests and their fringe areas in a holistic and sustainable way.

Q.76) Explanation:

https://edutap.learnyst.com/admin/products/all-questions/385022 54/71
3/29/23, 10:06 AM SchemesTap - January 2023 - Quiz-complete-test

The restructured NPDD Scheme is implemented from 2021-22 to 2025-26.

Q.77) Explanation:
Objectives of National Programme for Dairy Development:

·       To create and strengthen infrastructure for production of quality milk including cold chain
infrastructure linking the farmer to the consumer.

·     To create and strengthen infrastructure for procurement, processing and marketing of milk.

·        To create training infrastructure for training of dairy farmers.

·        To strengthen dairy cooperative societies/producers companies at village level.

·        To increase milk production by providing technical input services like cattle feed, and mineral
mixture etc.

·        To assist in rehabilitation of potentially viable milk federations/unions

Q.78) Explanation:
It is a Central Sector Scheme, implemented through the State Implementing Agency (SIA) i.e. State
Cooperative Dairy Federation with a budget allocation of Rs 1790 crore.

Q.79) Explanation:

·     The Nirbhaya Fund Framework provides for a non-lapsable corpus fund for safety and security of
women.

·     It is administered by the Department of Economic Affairs (DEA) of the Ministry of Finance (MoF) of


the Government of India.

·              It is a dedicated fund which can be utilized for projects specifically designed to improve the
safety and security of women. 

Q.80) Explanation:
Under the Nirbhaya Fund, one of the schemes namely “One Stop Centre (OSC) Scheme” is
implemented across the country since 1st April 2015.

 
https://edutap.learnyst.com/admin/products/all-questions/385022 55/71
3/29/23, 10:06 AM SchemesTap - January 2023 - Quiz-complete-test

Q.81) Explanation:
WHL will be universalised through short code 181 number by augmenting/using the infrastructure of
existing helplines working at the State/UT level.
 

Q.82) Explanation:

·     The funds are released in two installments of 40% and 60% (Statement A is incorrect).

·     It is a Central Sector Scheme (Statement B is correct).

·              The NESIDS will be over and above the existing schemes of Government of India and State
Governments of the NE Region (Statement C is correct).
 

Q.83) Explanation:
·     North East Special Infrastructure Development Scheme (NESIDS) was launched in the year 2017.

Q.84) Explanation:

·       Initially  a token amount of Rs. 10  lakh is released and balance amount of first installment are
released after receipt of letter of award of the work.

Q.85) Explanation:
The above paragraph mentions – One Stop Centre Scheme (OSC)

·       The OSC will support all women including girls below 18 years of age affected by violence,
irrespective of caste, class, religion, region, sexual orientation or marital status (Statement A is correct).

·     The scheme will be funded through Nirbhaya Fund (Statement B is correct).

·        The funds would be made available by Ministry of Women and Child Development to the District
Collector/District Magistrate directly (Statement C is incorrect).

https://edutap.learnyst.com/admin/products/all-questions/385022 56/71
3/29/23, 10:06 AM SchemesTap - January 2023 - Quiz-complete-test

Q.86) Explanation:
The above paragraph mentions – One Stop Centre Scheme (OSC)

Location Preferences for setting up of OSCs –

·        First preference: suitable and adequate accommodation with separate access having at least 5
rooms and carpet area of 132 sq.m. within a hospital / medical facility (staff quarter within or outside
the premises of the hospital may also be used).

·        Second preference: If it is not possible to locate accommodation within a hospital or medical
facility, then an existing Government/Semi Government institutions/Women Institutions/Swadhar
Grehs/ Working Women Hostels located within 2 km radius of the hospital/medical facility in the
district headquarter having adequate accommodation with separate access may be preferred for
operating the Centre.

·        Third preference: If existing accommodation is not feasible, the Centre may be constructed on
adequate land either within hospital/medical facility or within 2 km radius of the hospital/medical
facility, having at least an area of 300 sq.m. as identified by the concerned State Government (Carpet
area – 132 sq metre).

Q.87) Explanation:
Statement 4 is incorrect (Employment assistance is not provided under One Stop Centres)

Services provided by OSCs:

·        Emergency response and rescue services

·        Medical assistance

·        Assistance to women in lodging the FIR/NCR/DIR

·        Psycho- social support and counselling

·        Legal aid and counselling

·        Shelter

·        Video conferencing facility

Q.88) Explanation:
Funding Pattern:

·       Funding pattern under the scheme is in the ratio of 60:40 by the Central and State Governments
respectively (Option 2 is correct).

https://edutap.learnyst.com/admin/products/all-questions/385022 57/71
3/29/23, 10:06 AM SchemesTap - January 2023 - Quiz-complete-test

·       In case of North Eastern and Himalayan States, Central Assistance is provided in the ratio of 90:10
(Centre: State) and for Union Territories, the assistance is 100%.

Q.89) Explanation:
·     All statements A, B, C are correct with respect to this scheme.

Q.90) Explanation:
Financial Assistance under PKVY:

·       Under the programme, financial assistance of Rs 31000/ha/3 years is provided for


preparation/procurement of organic inputs such as bio/organic fertilisers, biopesticides, seeds etc. through
DBT (Statement A is correct).

·     Rs 8800/ha/3 years is provided for value addition and marketing that includes post-harvest management
practices like storage (Statement B is correct).

·        Financial assistance of Rs 3000/hectare for 3 years is provided for Cluster formation (of 20 ha) and
Capacity building including exposure visits, and trainings of field functionaries (Statement C is correct).

Q.91) Explanation:
·       The Production Linked Incentive (PLI) Scheme for Pharmaceuticals Sector is implemented by the
Ministry of Chemicals and Fertilizers (Statement A is correct).

·       The Production Linked Incentive (PLI) Scheme for Pharmaceuticals Sector was launched in the year
2021 (Statement B is correct).

·              Applicants are grouped based on Global Manufacturing Revenue (FY 2019-20) of pharmaceutical
goods (Statement C is incorrect)

o   Group A - more than or equal to Rs. 5,000 crore.

o   Group B - Between Rs. 500 (inclusive) crore and Rs. 5,000 crore.

o   Group C - Less than Rs. 500 crore

Q.92) Explanation:
·     Budgetary Outlay - Rs 15,000 crore.

·     The incentive allocation among the target groups is as follows:


https://edutap.learnyst.com/admin/products/all-questions/385022 58/71
3/29/23, 10:06 AM SchemesTap - January 2023 - Quiz-complete-test

o   Group A: Rs 11,000 crore

o   Group B: Rs 2,250 crore

o   Group C: Rs 1,750 crore

Q.93) Explanation:
·     Implementing Years - FY 2020-21 to 2028-29

·     The eligible products have been categorized into three categories.

o   Category 1

§  Biopharmaceuticals, Complex generic drugs, Patented drugs or drugs nearing patent expiry, Cell
based or gene therapy drugs, Orphan drugs, Special empty capsules like HPMC, Pullulan, enteric etc.,
Complex excipients, Phyto-pharmaceuticals, Other drugs as approved
o   Category 2

§  Active Pharmaceutical Ingredients / Key Starting Materials / Drug Intermediates

o   Category 3 (Drugs not covered under Category 1 and Category 2)

§  Repurposed drugs, Auto immune drugs, anti-cancer drugs, anti-diabetic drugs, anti-infective drugs,
cardiovascular drugs, psychotropic drugs and anti-retroviral drugs, In vitro diagnostic devices, Other drugs
as approved, Other drugs not manufactured in India

o   Rate of incentive will be 10% (of incremental sales value) for Category 1 and Category 2 products for
first four years, 8% for the fifth year and 6% for the sixth year of production under the scheme.

o   Rate of incentive will be 5% (of incremental sales value) for Category 3 products for first four years, 4%
for the fifth year and 3% for the sixth year of production under the scheme.

o   This will include the period for processing of applications (FY 2020-21), optional gestation period of one
year (FY 2021-22), incentive for 6 years and FY 2028-29 for disbursal of incentive for sales of FY 2027-28.

o    Incremental sales of a product mean sales of that product in a year over and above the sales of that
product in FY 2019-2020.

Q.94) Explanation:
·     The Production Linked Incentive (PLI) Scheme for Promotion of Domestic Manufacturing of Medical
Devices is implemented by the Ministry of Chemicals and Fertilizers (Statement A is correct).

·       The Production Linked Incentive (PLI) Scheme for Promotion of Domestic Manufacturing of Medical
Devices was launched in the year 2020 (Statement B is incorrect).

·        Selection Process (Statement C is correct)

https://edutap.learnyst.com/admin/products/all-questions/385022 59/71
3/29/23, 10:06 AM SchemesTap - January 2023 - Quiz-complete-test

o   Applicants will be evaluated and ranked basis factors including manufacturing turnover, Existing Patent /
Technology, R & D expenses, Existing ISO 13485 certification and regulatory approvals. 

§  A maximum of 28 applicants shall be selected under the Scheme basis score obtained as per the
Evaluation.

§  A maximum of 10 applicants shall be selected under each target segment.

§  A minimum of 3 applicants, if available, shall be selected under each Target Segment.

Q.95) Explanation:
·     Financial outlay of Rs. 3,420 cr. for the period 2020-21 to 2027-28.

Q.96) Explanation:
·     5% incentive will be provided on incremental sales (over Base Year: FY 2019-20) of medical devices
manufactured in India.

·     The incentives will be offered for a period of 5 years from FY 2020-21 to FY 2025-26.

·        Target Medical Devices Segments –

o   Cancer care / Radiotherapy

o   Radiology & Imaging and Nuclear Imaging Devices

o   Anesthetics & Cardio-Respiratory & Renal Care

o   All Implants

·        It will lead to expected incremental production of Rs. 68,437 crore over a period of five years.

·        The Schemes will lead to generation of additional employment of 33,750 jobs over a period of five
years.

Q.97) Explanation:
·       Pradhan Mantri Awaas Yojana - Gramin Scheme is implemented by the Ministry of Rural
Development (Statement A is correct).

·     Constructing 2.95 crore PMAY-G houses with all basic amenities by the year 2024. (Statement B is
incorrect).

·        The grants under the scheme are shared between the Centre and States in the ratio of 90:10 in case
of NE States, Himalayan States & Himalayan UTs. For all other States, funds are shared in the ratio of
60:40 by the Centre and the States. In cases of other UTs, entire funds are provided by the Centre.
https://edutap.learnyst.com/admin/products/all-questions/385022 60/71
3/29/23, 10:06 AM SchemesTap - January 2023 - Quiz-complete-test

(Statement C is correct).

Q.98) Explanation:
·     The minimum size of the house has been increase to 25 sq.mt (from 20 sq.mt) with a hygienic cooking
space.

Q.99) Explanation:
·       Unit assistance of Rs. 1.20 lakh in plain and Rs 1.30 lakh in hilly states, difficult areas and IAP
district.

·     Beneficiaries can also avail loan up to Rs. 70000 from financial institutions.

Q.100) Explanation:

·       Gati Shakti Digital Platform will bring 16 Ministries and 7 core infrastructure sectors together
for integrated planning and coordinated implementation of infrastructure connectivity projects through
real-time coordination between various ministries/departments (Statement A is correct).

·     Gati Shakti Digital Platform was launched in the year 2021 (Statement B is incorrect).

·        The scheme will be implemented for a duration of 4 years (Statement C is correct).

Q.101) Explanation:
·     It plans to establish 11 industrial corridors and two new defense corridors in Tamil Nadu and Uttar
Pradesh (Statement A is incorrect).

·     Implementation Framework


o    Empowered Group of Secretaries (EGOS) will be headed by Cabinet Secretary and will consist of
Secretaries of 18 ministries as members and Head of Logistics Division as Member Convenor (Statement B
is correct).

§  The EGOS has been mandated to review and monitor implementation of the PM GatiShakti NMP to
ensure logistics efficiency

o    Network Planning Group (NPG) – It will consist of heads of Network Planning wing of respective
infrastructure ministries and it will assist EGOS.
o   Technical Support Unit (TSU) – It will have domain experts from various infrastructure sectors.

https://edutap.learnyst.com/admin/products/all-questions/385022 61/71
3/29/23, 10:06 AM SchemesTap - January 2023 - Quiz-complete-test

·              A GIS based ERP system, will be developed in collaboration with BISAG-N (Bhaskaracharya
National Institute for Space Applications and Geo-informatics) (Statement C is correct).

Q.102) Explanation:
·     Pillars

o   Comprehensiveness

o   Prioritization

o   Dynamic

o   Optimization

o   Analytical

o   Synchronization

Q.103) Explanation:

·     This scheme has been included as a sub-scheme under Mission Shakti (Statement A is Correct).

·       Financial assistance of Rs. 5,000 are provided in TWO instalments. Number of instalments have
been reduced from 3 to 2 (Statement B is Incorrect).

·        Maternity Benefit Programme is being implemented in all districts of country in accordance with
the provision of the National Food Security Act, 2013 (Statement C is Correct).

Q.104) Explanation:
Eligible beneficiaries under PMMVY:

·     All pregnant and lactating women, excluding –

o   those in regular employment with Central Govt/ State Govt/ PSUs or those who have received similar
benefits earlier.

o   AWWs/ AWHs/ ASHA may also avail the benefits subject to fulfilment of scheme conditionalities.

·     Considering above factors, Option C (HR manager at ONGC Ltd) is not eligible.

Q.105) Explanation:

https://edutap.learnyst.com/admin/products/all-questions/385022 62/71
3/29/23, 10:06 AM SchemesTap - January 2023 - Quiz-complete-test

·     All the statements are correct with resect to the PMMVY, hence correct answer to this question is –
None of these

Q.106) Explanation:
Socio-economic empowerment of minorities with special emphasis on artisan communities, women,
and youth.

Q.107) Explanation:
One of components of the scheme aims to provide entrepreneurship and leadership support
exclusively to women wherein aspiring women entrepreneurs shall be provided 3 weeks (100 hours)
intensive training on an entrepreneurship development module.

Q.108) Explanation:
10% of these women trained as entrepreneurs shall also be selected to become Business Mentors,
based on their leadership qualities. These Business Mentors/ correspondents shall be provided a 30
days (240 hours) training.

Q.109) Explanation:

·       Rashtriya Gokul Mission (RGM) is implemented by the Department of Animal Husbandry and
Dairying (Statement A is correct).

·     Rashtriya Gokul Mission (RGM) was launched in the year 2014 (Statement B is incorrect).

·        This scheme has been continued for 5 years, from FY 2021-22 to 2025-26 (Statement C is correct).

Q.110) Explanation:
·     All the components of Scheme will be implemented on 100% grant-in-aid basis except the components
of

o   Accelerated breed improvement programme under the component subsidy of Rs 5000 per IVF pregnancy
will be made available to participating farmers as GoI share.

https://edutap.learnyst.com/admin/products/all-questions/385022 63/71
3/29/23, 10:06 AM SchemesTap - January 2023 - Quiz-complete-test

o   Promoting sex sorted semen under the component subsidy upto 50% of the cost of sex sorted semen will
be made available to participating farmers.

o   Establishment of breed multiplication farm under the component subsidy upto 50% of the capital cost
maximum upto Rs.2.00 crore of the project will be made available to entrepreneur.

Q.111) Explanation:
·     Components of RGM

o   Availability of High genetic Merit Germplasm:

o   Extension of Artificial Insemination Network

o   Development and Conservation of indigenous Breeds

o   Skill Development

o   Farmers Awareness

o   Research Development and Innovation in Bovine Breeding

Q.112) Explanation:

·     Rashtriya Gram Swaraj Abhiyan (RGSA)is implemented by the Ministry of Panchayat Raj (Statement
A is correct).

·     Rashtriya Gram Swaraj Abhiyan (RGSA)was launched in the year 2018 (Statement B is correct).

·        This scheme is a centrally sponsored scheme (Statement C is correct).

Q.113) Explanation:
·     Funding - Total financial outlay of the scheme is Rs. 5911 crore with the Central Share of Rs. 3700
crore and State Share of Rs.2211 crore.

Cabinet approves continuation of revamped Centrally Sponsored Scheme of Rashtriya Gram Swaraj
Abhiyan (RGSA) from 01.04.2022 to 31.03.2026

Q.114) Explanation:

https://edutap.learnyst.com/admin/products/all-questions/385022 64/71
3/29/23, 10:06 AM SchemesTap - January 2023 - Quiz-complete-test

·     Aim - Strengthening of Panchayati Raj Institutions for achieving Sustainable Development Goals with
main thrust on convergence with Mission Antyodaya and emphasis on strengthening Panchayati Raj
Institutions in 117 Aspirational districts.

Q.115) Explanation:
·     Rashtriya Krishi Vikas Yojana (RKVY) is implemented by the Ministry of Agriculture and Farmers
Welfare (Statement A is correct).

·     Rashtriya Krishi Vikas Yojana (RKVY) was launched in the year 2007 (Statement B is correct).

·        Funding Pattern (Statement C is correct)

o    Till 2013-14, the scheme was implemented as an Additional Central Assistance (ACA) to State Plan
Scheme with 100% central assistance.
o   It was converted into a Centrally Sponsored Scheme in 2014-15 also with 100% central assistance.

o   Since 2015-16, the funding pattern of the scheme has been altered in the ratio of 60:40 between Centre
and States (90:10 for North Eastern States and Himalayan States).

o   For Union Territories the funding pattern is 100 % central grant.

Q.116) Explanation:
·     Allied sectors covered under the scheme -
o   Crop Husbandry (including Horticulture)

o   Animal Husbandry, Dairy Development and Fisheries

o   Agricultural Research and Education

o   Agricultural Marketing

o   Food storage and Warehousing

o   Soil and Water Conservation

o   Agricultural Financial Institutions

o   Other Agriculture Programmes and Cooperation

Q.117) Explanation:
·     Sub-Schemes under RKVY

o   Bringing Green Revolution to Eastern India


https://edutap.learnyst.com/admin/products/all-questions/385022 65/71
3/29/23, 10:06 AM SchemesTap - January 2023 - Quiz-complete-test

o   Crop Diversification Program

o   Additional Fodder Development Programme

o   Reclamation of Problem Soil

o   Area Expansion of Cashew

o   Pilot Intervention for Most Vulnerable Drought Prone Districts

o   National Saffron Mission

o   Integrated Development of 60,000 Pulses Village in Rainfed Areas 

o   Special initiatives for pulses and oilseeds in dry land areas of 60000 villages

Q.118) Explanation:
Eligibility:
Indian citizen of any age

Has 40% disability or more

Monthly income not more than Rs 30000

Must not have received assistance during the last 3 years and for children (below age 12 years) in last 1
year.
 

Q.119) Explanation:
Financial Assistance

·     Full financial assistance for aids/appliances costing upto Rs. 15,000.

·    Financial assistance upto Rs. 15000 for appliances costing between Rs. 15,001 to Rs. 30000.

·       Government of India shall bear 50% of cost of items costing above Rs. 30,001/, except Cochlear
Implant and Motorized Tricycle.

Q.120) Explanation:
Aim:

·     To improve the financial position of the auqaf and the Waqf Boards (Statement A is correct)

·     To enable them to enlarge the area of their welfare work

·        To protect vacant Waqf land from encroachers (Statement B is incorrect)


https://edutap.learnyst.com/admin/products/all-questions/385022 66/71
3/29/23, 10:06 AM SchemesTap - January 2023 - Quiz-complete-test

·        To develop economically viable projects on these properties for generating more income and /or
to widen welfare activities (Statement C is correct)

Q.121) Explanation:
·         All WBs/ registered waqf institutions, which are in full possession of property to be developed
and in the opinion of CWC are not insolvent or otherwise suitable, shall be eligible for interest free
loan.

·         Maximum interest free loan payable will be Rs 2 crore or 75% of the estimated cost of the
project (excluding the value of the land), whichever is less.
 

Q.122) Explanation:
The loan shall be ordinarily repayable to the CWC in 16 half-yearly installments, with the moratorium
of one year after the final installment of loan is disbursed.

Q.123) Explanation:
·       Skill India Mission is implemented by the Ministry Skill Development and Entrepreneurship
(Statement A is correct).

·     Skill India Mission was launched in the year 2015 (Statement B is correct).

·              It aims to provide market-relevant skills training to more than 40 crore young people in the
country by the year 2022. (Statement C is incorrect)

Q.124) Explanation:

·     Ministry imparts employable skills to the youth through long term and short term training.

o   For short term training - Ministry is implementing Pradhan Mantri Kaushal Vikas Yojana (PMKVY
2.0) and Jan Shikshan Sansthan (JSS).

o    Long Term Training is imparted through Industrial Training Institutes under Craftsmen Training
Scheme.

·       The mission offers training, guidance, and support for all traditional types of employment like
weavers, cobblers, carpenters, welders, masons, blacksmiths, nurses, etc.

https://edutap.learnyst.com/admin/products/all-questions/385022 67/71
3/29/23, 10:06 AM SchemesTap - January 2023 - Quiz-complete-test

·        Training imparted would be of international standards so that India’s youth get jobs not only in India
but also abroad where there is demand.

Q.125) Explanation:
·     Sub-schemes under Skill India

o   National Skill Development Mission (NSDM)

o   Pradhan Mantri Kaushal Vikas Yojana (PMKVY)

o   Indian Skill Development Service

o   National Policy for Skill Development and Entrepreneurship 2015

o   Skill Loan Scheme

o   Pradhan Mantri Yuva Udyamita Vikas Abhiyan (PM-YUVA)

o   Apprenticeship Protsahan Yojana

Q.126) Explanation:
·     Strengthening of Pharmaceutical Industry (SPI)' Scheme aims to strengthen the existing infrastructure
facilities in order to make India a global leader in the Pharma Sector.

Q.127) Explanation:
·        An outlay of ₹21.5 crore has been earmarked for the sub-scheme for the scheme period of five
years.

·              Pharmaceutical & Medical Devices Promotion and Development Scheme (PMPDS) to facilitate
growth and development of Pharmaceutical and Medical Devices Sectors through study/survey reports,
awareness programs, creation of database, and promotion of industry.

·        Knowledge and awareness about the Pharmaceutical and MedTech Industry will be promoted. This
will be done by undertaking studies, building databases and bringing industry leaders, academia and
policymakers together to share their knowledge and experience for the overall development of the Pharma
and Medical Devices sector.

Q.128) Explanation:
·              Pharmaceutical Technology Upgradation Assistance Scheme (PTUAS) to facilitate MSMEs of a
proven track record to meet national and international regulatory standards
https://edutap.learnyst.com/admin/products/all-questions/385022 68/71
3/29/23, 10:06 AM SchemesTap - January 2023 - Quiz-complete-test

·        Support for SME Industries is proposed, either through up to a maximum of 5% per annum (6% in
case of units owned and managed by SC/STs) of interest subvention or through Credit linked Capital
subsidy of 10%.

·        In both cases, the loan supported under this is to a limit of 10 Crores and the eligible components
of the loan has been listed in the scheme guidelines. An outlay of ₹300 crore has been earmarked.

Q.129) Explanation:
All three statements are correct with respect to scheme –

·        Under SBM(G) Phase 2, Gobardhan initiative has been launched to convert bio-waste into biogas and
bio slurry.

·        Under SBM(G) Phase-II, financial assistance of up to Rs. 50 lakh per district is available for setting up
of community and cluster model biogas plants.

·        Geo-tagging of toilets has been made mandatory under SBM(G).

Q.130) Explanation:
SBM-U:

·     It is operated as a Centrally Sponsored Scheme (Statement A is incorrect).

·     Tenure - From 2020-21 till 2024-25 (Statement B is correct)

·     Financial outlay for this scheme is Rs. 1,40,881 crores (Statement C is correct).

Q.131) Explanation:
·        For Swachh Bharat Mission-Grameen, Rs 7,192 Crore has been allocated in the budget for the
year 2022-23.

 
 

Q.132) Explanation:
Objectives of Swachh Bharat Mission (Grameen) Phase II

·        Open defecation free behaviours are sustained and no one is left behind.

https://edutap.learnyst.com/admin/products/all-questions/385022 69/71
3/29/23, 10:06 AM SchemesTap - January 2023 - Quiz-complete-test

·        Solid and liquid waste management facilities are accessible and reinforcing ODF behaviours and focus
on providing interventions for safe management of solid and liquid waste in villages

·              To encourage cost effective and appropriate technologies for ecologically safe and sustainable
sanitation.

·        To develop, wherever required, community managed sanitation systems focusing on scientific Solid &
Liquid Waste Management systems for overall cleanliness in the rural areas.

·        To create significant positive impact on gender and promote social inclusion by improving sanitation
especially in marginalized communities.

Q.133) Explanation:
·        Under the Concessional Credit/Weaver MUDRA Scheme, Margin Money assistance at 20% of loan
amount, loan at 6% interest rate and Credit Guarantee for a period of 3 years is provided to handloom
weavers/weaver entrepreneurs across the country. However, the Government of India interest subvention is
capped at 7%.

Q.134) Explanation:
Financial assistance is provided as follows -
·        Margin money assistance

·        @ 20% of loan amount, subject to maximum of Rs. 25,000 per weaver,

·              @ 20% of loan amount, subject to maximum of Rs. 20.00 lakh (@ Rs. 2.00 lakh for every 100
weaver/worker) per handloom organization,

·        Interest subvention upto 7% for 3 years

·        Credit Guarantee on loans for 3 years

Q.135) Explanation:
·     For timely transfer of financial assistance, Handloom Weaver MUDRA Portal has been developed in
association with Punjab National Bank to cut down delay in disbursement of funds for margin money and
interest subvention.

·     The margin money assistance is transferred directly to loan account of weaver and interest subvention is
transferred to banks through the portal.

 
https://edutap.learnyst.com/admin/products/all-questions/385022 70/71
3/29/23, 10:06 AM SchemesTap - January 2023 - Quiz-complete-test

https://edutap.learnyst.com/admin/products/all-questions/385022 71/71

You might also like